Inverse trigonometric function identity doubt: $tan^{-1}x+tan^{-1}y =-pi+tan^{-1}left(frac{x+y}{1-xy}right)$,...












3












$begingroup$


According to my book




$$tan^{-1}x+tan^{-1}y =-pi+tan^{-1}left(frac{x+y}{1-xy}right)$$
when $x<0$, $y<0$, and $xy>1$.




I can't understand one thing out here that when the above mentioned conditions on $x$ and $y$ are followed then the denominator of the argument of $tan^{-1}(1-xy)$ become negative while the numerator too becomes negative and $x$ and $y$ both are less than zero. Now as both the numerator and denominator are negative the arguments i.e $left(frac{x+y}{1-xy}right)$becomes positive overall.



Now why do we add $pi$ to the expression when we are already having a positive argument which can be found in the first quadrant which is found in principal range. Now is it because we can also find the postive tangent function in third quadrant also? If this is so, why has this been mentioned up as a separate identity rather than another solution?










share|cite|improve this question











$endgroup$








  • 1




    $begingroup$
    The problem is that $tan^{-1}x+tan^{-1}y$ "overflows". Meaning, it is not necessarily in the usual range $(-pi/2,pi/2)$ of $arctan$. See what happens when, for example, $x=y=-sqrt3$.
    $endgroup$
    – Jyrki Lahtonen
    Jun 23 '16 at 20:01






  • 1




    $begingroup$
    For the right values of $x,y, tan^{-1} x + tan^{-1} y$ are less than $-pi$
    $endgroup$
    – Doug M
    Jun 23 '16 at 20:05












  • $begingroup$
    Indeed $${{tan }^{-1}}x+{{tan }^{-1}}y=left{ begin{align} & qquadquad{{tan }^{-1}}frac{x+y}{1-xy}quad,quad xy<1 \ & quadpi+{{tan }^{-1}}frac{x+y}{1-xy}quad,quad xy>1quad,quad x>0 \ & -pi+{{tan }^{-1}}frac{x+y}{1-xy}quad,quad xy>1quad,quad x<0 \ end{align} right.$$
    $endgroup$
    – Behrouz Maleki
    Jun 23 '16 at 20:14
















3












$begingroup$


According to my book




$$tan^{-1}x+tan^{-1}y =-pi+tan^{-1}left(frac{x+y}{1-xy}right)$$
when $x<0$, $y<0$, and $xy>1$.




I can't understand one thing out here that when the above mentioned conditions on $x$ and $y$ are followed then the denominator of the argument of $tan^{-1}(1-xy)$ become negative while the numerator too becomes negative and $x$ and $y$ both are less than zero. Now as both the numerator and denominator are negative the arguments i.e $left(frac{x+y}{1-xy}right)$becomes positive overall.



Now why do we add $pi$ to the expression when we are already having a positive argument which can be found in the first quadrant which is found in principal range. Now is it because we can also find the postive tangent function in third quadrant also? If this is so, why has this been mentioned up as a separate identity rather than another solution?










share|cite|improve this question











$endgroup$








  • 1




    $begingroup$
    The problem is that $tan^{-1}x+tan^{-1}y$ "overflows". Meaning, it is not necessarily in the usual range $(-pi/2,pi/2)$ of $arctan$. See what happens when, for example, $x=y=-sqrt3$.
    $endgroup$
    – Jyrki Lahtonen
    Jun 23 '16 at 20:01






  • 1




    $begingroup$
    For the right values of $x,y, tan^{-1} x + tan^{-1} y$ are less than $-pi$
    $endgroup$
    – Doug M
    Jun 23 '16 at 20:05












  • $begingroup$
    Indeed $${{tan }^{-1}}x+{{tan }^{-1}}y=left{ begin{align} & qquadquad{{tan }^{-1}}frac{x+y}{1-xy}quad,quad xy<1 \ & quadpi+{{tan }^{-1}}frac{x+y}{1-xy}quad,quad xy>1quad,quad x>0 \ & -pi+{{tan }^{-1}}frac{x+y}{1-xy}quad,quad xy>1quad,quad x<0 \ end{align} right.$$
    $endgroup$
    – Behrouz Maleki
    Jun 23 '16 at 20:14














3












3








3


6



$begingroup$


According to my book




$$tan^{-1}x+tan^{-1}y =-pi+tan^{-1}left(frac{x+y}{1-xy}right)$$
when $x<0$, $y<0$, and $xy>1$.




I can't understand one thing out here that when the above mentioned conditions on $x$ and $y$ are followed then the denominator of the argument of $tan^{-1}(1-xy)$ become negative while the numerator too becomes negative and $x$ and $y$ both are less than zero. Now as both the numerator and denominator are negative the arguments i.e $left(frac{x+y}{1-xy}right)$becomes positive overall.



Now why do we add $pi$ to the expression when we are already having a positive argument which can be found in the first quadrant which is found in principal range. Now is it because we can also find the postive tangent function in third quadrant also? If this is so, why has this been mentioned up as a separate identity rather than another solution?










share|cite|improve this question











$endgroup$




According to my book




$$tan^{-1}x+tan^{-1}y =-pi+tan^{-1}left(frac{x+y}{1-xy}right)$$
when $x<0$, $y<0$, and $xy>1$.




I can't understand one thing out here that when the above mentioned conditions on $x$ and $y$ are followed then the denominator of the argument of $tan^{-1}(1-xy)$ become negative while the numerator too becomes negative and $x$ and $y$ both are less than zero. Now as both the numerator and denominator are negative the arguments i.e $left(frac{x+y}{1-xy}right)$becomes positive overall.



Now why do we add $pi$ to the expression when we are already having a positive argument which can be found in the first quadrant which is found in principal range. Now is it because we can also find the postive tangent function in third quadrant also? If this is so, why has this been mentioned up as a separate identity rather than another solution?







trigonometry






share|cite|improve this question















share|cite|improve this question













share|cite|improve this question




share|cite|improve this question








edited Jun 23 '16 at 20:00









Blue

48k870153




48k870153










asked Jun 23 '16 at 19:56









Harsh SharmaHarsh Sharma

1,292625




1,292625








  • 1




    $begingroup$
    The problem is that $tan^{-1}x+tan^{-1}y$ "overflows". Meaning, it is not necessarily in the usual range $(-pi/2,pi/2)$ of $arctan$. See what happens when, for example, $x=y=-sqrt3$.
    $endgroup$
    – Jyrki Lahtonen
    Jun 23 '16 at 20:01






  • 1




    $begingroup$
    For the right values of $x,y, tan^{-1} x + tan^{-1} y$ are less than $-pi$
    $endgroup$
    – Doug M
    Jun 23 '16 at 20:05












  • $begingroup$
    Indeed $${{tan }^{-1}}x+{{tan }^{-1}}y=left{ begin{align} & qquadquad{{tan }^{-1}}frac{x+y}{1-xy}quad,quad xy<1 \ & quadpi+{{tan }^{-1}}frac{x+y}{1-xy}quad,quad xy>1quad,quad x>0 \ & -pi+{{tan }^{-1}}frac{x+y}{1-xy}quad,quad xy>1quad,quad x<0 \ end{align} right.$$
    $endgroup$
    – Behrouz Maleki
    Jun 23 '16 at 20:14














  • 1




    $begingroup$
    The problem is that $tan^{-1}x+tan^{-1}y$ "overflows". Meaning, it is not necessarily in the usual range $(-pi/2,pi/2)$ of $arctan$. See what happens when, for example, $x=y=-sqrt3$.
    $endgroup$
    – Jyrki Lahtonen
    Jun 23 '16 at 20:01






  • 1




    $begingroup$
    For the right values of $x,y, tan^{-1} x + tan^{-1} y$ are less than $-pi$
    $endgroup$
    – Doug M
    Jun 23 '16 at 20:05












  • $begingroup$
    Indeed $${{tan }^{-1}}x+{{tan }^{-1}}y=left{ begin{align} & qquadquad{{tan }^{-1}}frac{x+y}{1-xy}quad,quad xy<1 \ & quadpi+{{tan }^{-1}}frac{x+y}{1-xy}quad,quad xy>1quad,quad x>0 \ & -pi+{{tan }^{-1}}frac{x+y}{1-xy}quad,quad xy>1quad,quad x<0 \ end{align} right.$$
    $endgroup$
    – Behrouz Maleki
    Jun 23 '16 at 20:14








1




1




$begingroup$
The problem is that $tan^{-1}x+tan^{-1}y$ "overflows". Meaning, it is not necessarily in the usual range $(-pi/2,pi/2)$ of $arctan$. See what happens when, for example, $x=y=-sqrt3$.
$endgroup$
– Jyrki Lahtonen
Jun 23 '16 at 20:01




$begingroup$
The problem is that $tan^{-1}x+tan^{-1}y$ "overflows". Meaning, it is not necessarily in the usual range $(-pi/2,pi/2)$ of $arctan$. See what happens when, for example, $x=y=-sqrt3$.
$endgroup$
– Jyrki Lahtonen
Jun 23 '16 at 20:01




1




1




$begingroup$
For the right values of $x,y, tan^{-1} x + tan^{-1} y$ are less than $-pi$
$endgroup$
– Doug M
Jun 23 '16 at 20:05






$begingroup$
For the right values of $x,y, tan^{-1} x + tan^{-1} y$ are less than $-pi$
$endgroup$
– Doug M
Jun 23 '16 at 20:05














$begingroup$
Indeed $${{tan }^{-1}}x+{{tan }^{-1}}y=left{ begin{align} & qquadquad{{tan }^{-1}}frac{x+y}{1-xy}quad,quad xy<1 \ & quadpi+{{tan }^{-1}}frac{x+y}{1-xy}quad,quad xy>1quad,quad x>0 \ & -pi+{{tan }^{-1}}frac{x+y}{1-xy}quad,quad xy>1quad,quad x<0 \ end{align} right.$$
$endgroup$
– Behrouz Maleki
Jun 23 '16 at 20:14




$begingroup$
Indeed $${{tan }^{-1}}x+{{tan }^{-1}}y=left{ begin{align} & qquadquad{{tan }^{-1}}frac{x+y}{1-xy}quad,quad xy<1 \ & quadpi+{{tan }^{-1}}frac{x+y}{1-xy}quad,quad xy>1quad,quad x>0 \ & -pi+{{tan }^{-1}}frac{x+y}{1-xy}quad,quad xy>1quad,quad x<0 \ end{align} right.$$
$endgroup$
– Behrouz Maleki
Jun 23 '16 at 20:14










3 Answers
3






active

oldest

votes


















1












$begingroup$

Consider $y$ constant and the functions
$$
f(x)=arctan x+arctan y
qquad
g(x)=-pi+arctanfrac{x+y}{1-xy}
$$
Then $f'(x)=1/(1+x^2)$, whereas
$$
g'(x)=frac{1}{1+dfrac{(x+y)^2}{(1-xy)^2}}frac{1-xy+y(x+y)}{(1-xy)^2}=
frac{1+y^2}{1+x^2+y^2+x^2y^2}=frac{1}{1+x^2}
$$
Therefore the two functions differ by a constant in every connected component of their domain.



Suppose $x<0$, $y<0$ and $xy>1$. Then we can consider the limit at $-infty$ of $f$ and $g$:
$$
lim_{xto-infty}f(x)=-frac{pi}{2}+arctan y
$$
while
$$
lim_{xto-infty}g(x)=-pi+arctanfrac{1}{-y}=-pi+frac{pi}{2}+arctan y
$$
due to
$$
arctan y+arctanfrac{1}{y}=-frac{pi}{2}
$$
for $y<0$.



Thus $f(x)=g(x)$ in the stated domain.






share|cite|improve this answer









$endgroup$





















    1












    $begingroup$

    From the Article $240,$ Ex$-5$ of Plane Trigonometry(by Loney),



    $$arctan x+arctan y=begin{cases} arctanfrac{x+y}{1-xy} &mbox{if } xy<1\ pi+arctanfrac{x+y}{1-xy} & mbox{if } xy>1\text{sign}(x)cdotdfracpi2 & mbox{if } xy=1end{cases} $$






    share|cite|improve this answer









    $endgroup$













    • $begingroup$
      I don't get what Loney is trying to tell in his last statement. He says that $gamma$ is a negative angle and $tan(pi + gamma)= tan gamma$ but $gamma$ is still negative :/ How does that make a difference? Could you explain through an example..
      $endgroup$
      – Abcd
      Feb 8 '18 at 12:27



















    1












    $begingroup$

    This is not a complete answer, but rather a response to the following doubt:




    Now why do we add π to the expression when we are already having a positive argument ...




    Actually, we subtract $pi.$ But how are we to understand intuitively that this is something we might want to do?



    As you observed, under the given conditions on $x$ and $y,$ we know that
    $frac{x+y}{1-xy} > 0$ and therefore
    $tan^{-1}left(frac{x+y}{1-xy}right) > 0$ as well.



    But the given conditions include $x< 0$ and $y< 0$, from which it follows that
    $tan^{-1} x < 0,$ that $tan^{-1} y < 0,$ and that
    $tan^{-1} x + tan^{-1} y < 0.$



    We cannot have an equation with a negative number on the left and a positive number on the right, can we?
    But we can add or subtract something on the left or right in order to make the two sides equal after all.
    What is to be shown then is that the thing to add or subtract is a constant for all $x$ and $y$ that satisfy the given conditions,
    and that subtracting the particular constant $pi$ from the right side will make the equation satisfied.






    share|cite|improve this answer









    $endgroup$













      Your Answer





      StackExchange.ifUsing("editor", function () {
      return StackExchange.using("mathjaxEditing", function () {
      StackExchange.MarkdownEditor.creationCallbacks.add(function (editor, postfix) {
      StackExchange.mathjaxEditing.prepareWmdForMathJax(editor, postfix, [["$", "$"], ["\\(","\\)"]]);
      });
      });
      }, "mathjax-editing");

      StackExchange.ready(function() {
      var channelOptions = {
      tags: "".split(" "),
      id: "69"
      };
      initTagRenderer("".split(" "), "".split(" "), channelOptions);

      StackExchange.using("externalEditor", function() {
      // Have to fire editor after snippets, if snippets enabled
      if (StackExchange.settings.snippets.snippetsEnabled) {
      StackExchange.using("snippets", function() {
      createEditor();
      });
      }
      else {
      createEditor();
      }
      });

      function createEditor() {
      StackExchange.prepareEditor({
      heartbeatType: 'answer',
      autoActivateHeartbeat: false,
      convertImagesToLinks: true,
      noModals: true,
      showLowRepImageUploadWarning: true,
      reputationToPostImages: 10,
      bindNavPrevention: true,
      postfix: "",
      imageUploader: {
      brandingHtml: "Powered by u003ca class="icon-imgur-white" href="https://imgur.com/"u003eu003c/au003e",
      contentPolicyHtml: "User contributions licensed under u003ca href="https://creativecommons.org/licenses/by-sa/3.0/"u003ecc by-sa 3.0 with attribution requiredu003c/au003e u003ca href="https://stackoverflow.com/legal/content-policy"u003e(content policy)u003c/au003e",
      allowUrls: true
      },
      noCode: true, onDemand: true,
      discardSelector: ".discard-answer"
      ,immediatelyShowMarkdownHelp:true
      });


      }
      });














      draft saved

      draft discarded


















      StackExchange.ready(
      function () {
      StackExchange.openid.initPostLogin('.new-post-login', 'https%3a%2f%2fmath.stackexchange.com%2fquestions%2f1837410%2finverse-trigonometric-function-identity-doubt-tan-1x-tan-1y-pi-tan%23new-answer', 'question_page');
      }
      );

      Post as a guest















      Required, but never shown

























      3 Answers
      3






      active

      oldest

      votes








      3 Answers
      3






      active

      oldest

      votes









      active

      oldest

      votes






      active

      oldest

      votes









      1












      $begingroup$

      Consider $y$ constant and the functions
      $$
      f(x)=arctan x+arctan y
      qquad
      g(x)=-pi+arctanfrac{x+y}{1-xy}
      $$
      Then $f'(x)=1/(1+x^2)$, whereas
      $$
      g'(x)=frac{1}{1+dfrac{(x+y)^2}{(1-xy)^2}}frac{1-xy+y(x+y)}{(1-xy)^2}=
      frac{1+y^2}{1+x^2+y^2+x^2y^2}=frac{1}{1+x^2}
      $$
      Therefore the two functions differ by a constant in every connected component of their domain.



      Suppose $x<0$, $y<0$ and $xy>1$. Then we can consider the limit at $-infty$ of $f$ and $g$:
      $$
      lim_{xto-infty}f(x)=-frac{pi}{2}+arctan y
      $$
      while
      $$
      lim_{xto-infty}g(x)=-pi+arctanfrac{1}{-y}=-pi+frac{pi}{2}+arctan y
      $$
      due to
      $$
      arctan y+arctanfrac{1}{y}=-frac{pi}{2}
      $$
      for $y<0$.



      Thus $f(x)=g(x)$ in the stated domain.






      share|cite|improve this answer









      $endgroup$


















        1












        $begingroup$

        Consider $y$ constant and the functions
        $$
        f(x)=arctan x+arctan y
        qquad
        g(x)=-pi+arctanfrac{x+y}{1-xy}
        $$
        Then $f'(x)=1/(1+x^2)$, whereas
        $$
        g'(x)=frac{1}{1+dfrac{(x+y)^2}{(1-xy)^2}}frac{1-xy+y(x+y)}{(1-xy)^2}=
        frac{1+y^2}{1+x^2+y^2+x^2y^2}=frac{1}{1+x^2}
        $$
        Therefore the two functions differ by a constant in every connected component of their domain.



        Suppose $x<0$, $y<0$ and $xy>1$. Then we can consider the limit at $-infty$ of $f$ and $g$:
        $$
        lim_{xto-infty}f(x)=-frac{pi}{2}+arctan y
        $$
        while
        $$
        lim_{xto-infty}g(x)=-pi+arctanfrac{1}{-y}=-pi+frac{pi}{2}+arctan y
        $$
        due to
        $$
        arctan y+arctanfrac{1}{y}=-frac{pi}{2}
        $$
        for $y<0$.



        Thus $f(x)=g(x)$ in the stated domain.






        share|cite|improve this answer









        $endgroup$
















          1












          1








          1





          $begingroup$

          Consider $y$ constant and the functions
          $$
          f(x)=arctan x+arctan y
          qquad
          g(x)=-pi+arctanfrac{x+y}{1-xy}
          $$
          Then $f'(x)=1/(1+x^2)$, whereas
          $$
          g'(x)=frac{1}{1+dfrac{(x+y)^2}{(1-xy)^2}}frac{1-xy+y(x+y)}{(1-xy)^2}=
          frac{1+y^2}{1+x^2+y^2+x^2y^2}=frac{1}{1+x^2}
          $$
          Therefore the two functions differ by a constant in every connected component of their domain.



          Suppose $x<0$, $y<0$ and $xy>1$. Then we can consider the limit at $-infty$ of $f$ and $g$:
          $$
          lim_{xto-infty}f(x)=-frac{pi}{2}+arctan y
          $$
          while
          $$
          lim_{xto-infty}g(x)=-pi+arctanfrac{1}{-y}=-pi+frac{pi}{2}+arctan y
          $$
          due to
          $$
          arctan y+arctanfrac{1}{y}=-frac{pi}{2}
          $$
          for $y<0$.



          Thus $f(x)=g(x)$ in the stated domain.






          share|cite|improve this answer









          $endgroup$



          Consider $y$ constant and the functions
          $$
          f(x)=arctan x+arctan y
          qquad
          g(x)=-pi+arctanfrac{x+y}{1-xy}
          $$
          Then $f'(x)=1/(1+x^2)$, whereas
          $$
          g'(x)=frac{1}{1+dfrac{(x+y)^2}{(1-xy)^2}}frac{1-xy+y(x+y)}{(1-xy)^2}=
          frac{1+y^2}{1+x^2+y^2+x^2y^2}=frac{1}{1+x^2}
          $$
          Therefore the two functions differ by a constant in every connected component of their domain.



          Suppose $x<0$, $y<0$ and $xy>1$. Then we can consider the limit at $-infty$ of $f$ and $g$:
          $$
          lim_{xto-infty}f(x)=-frac{pi}{2}+arctan y
          $$
          while
          $$
          lim_{xto-infty}g(x)=-pi+arctanfrac{1}{-y}=-pi+frac{pi}{2}+arctan y
          $$
          due to
          $$
          arctan y+arctanfrac{1}{y}=-frac{pi}{2}
          $$
          for $y<0$.



          Thus $f(x)=g(x)$ in the stated domain.







          share|cite|improve this answer












          share|cite|improve this answer



          share|cite|improve this answer










          answered Jun 23 '16 at 21:10









          egregegreg

          181k1485203




          181k1485203























              1












              $begingroup$

              From the Article $240,$ Ex$-5$ of Plane Trigonometry(by Loney),



              $$arctan x+arctan y=begin{cases} arctanfrac{x+y}{1-xy} &mbox{if } xy<1\ pi+arctanfrac{x+y}{1-xy} & mbox{if } xy>1\text{sign}(x)cdotdfracpi2 & mbox{if } xy=1end{cases} $$






              share|cite|improve this answer









              $endgroup$













              • $begingroup$
                I don't get what Loney is trying to tell in his last statement. He says that $gamma$ is a negative angle and $tan(pi + gamma)= tan gamma$ but $gamma$ is still negative :/ How does that make a difference? Could you explain through an example..
                $endgroup$
                – Abcd
                Feb 8 '18 at 12:27
















              1












              $begingroup$

              From the Article $240,$ Ex$-5$ of Plane Trigonometry(by Loney),



              $$arctan x+arctan y=begin{cases} arctanfrac{x+y}{1-xy} &mbox{if } xy<1\ pi+arctanfrac{x+y}{1-xy} & mbox{if } xy>1\text{sign}(x)cdotdfracpi2 & mbox{if } xy=1end{cases} $$






              share|cite|improve this answer









              $endgroup$













              • $begingroup$
                I don't get what Loney is trying to tell in his last statement. He says that $gamma$ is a negative angle and $tan(pi + gamma)= tan gamma$ but $gamma$ is still negative :/ How does that make a difference? Could you explain through an example..
                $endgroup$
                – Abcd
                Feb 8 '18 at 12:27














              1












              1








              1





              $begingroup$

              From the Article $240,$ Ex$-5$ of Plane Trigonometry(by Loney),



              $$arctan x+arctan y=begin{cases} arctanfrac{x+y}{1-xy} &mbox{if } xy<1\ pi+arctanfrac{x+y}{1-xy} & mbox{if } xy>1\text{sign}(x)cdotdfracpi2 & mbox{if } xy=1end{cases} $$






              share|cite|improve this answer









              $endgroup$



              From the Article $240,$ Ex$-5$ of Plane Trigonometry(by Loney),



              $$arctan x+arctan y=begin{cases} arctanfrac{x+y}{1-xy} &mbox{if } xy<1\ pi+arctanfrac{x+y}{1-xy} & mbox{if } xy>1\text{sign}(x)cdotdfracpi2 & mbox{if } xy=1end{cases} $$







              share|cite|improve this answer












              share|cite|improve this answer



              share|cite|improve this answer










              answered Jun 24 '16 at 4:57









              lab bhattacharjeelab bhattacharjee

              225k15157275




              225k15157275












              • $begingroup$
                I don't get what Loney is trying to tell in his last statement. He says that $gamma$ is a negative angle and $tan(pi + gamma)= tan gamma$ but $gamma$ is still negative :/ How does that make a difference? Could you explain through an example..
                $endgroup$
                – Abcd
                Feb 8 '18 at 12:27


















              • $begingroup$
                I don't get what Loney is trying to tell in his last statement. He says that $gamma$ is a negative angle and $tan(pi + gamma)= tan gamma$ but $gamma$ is still negative :/ How does that make a difference? Could you explain through an example..
                $endgroup$
                – Abcd
                Feb 8 '18 at 12:27
















              $begingroup$
              I don't get what Loney is trying to tell in his last statement. He says that $gamma$ is a negative angle and $tan(pi + gamma)= tan gamma$ but $gamma$ is still negative :/ How does that make a difference? Could you explain through an example..
              $endgroup$
              – Abcd
              Feb 8 '18 at 12:27




              $begingroup$
              I don't get what Loney is trying to tell in his last statement. He says that $gamma$ is a negative angle and $tan(pi + gamma)= tan gamma$ but $gamma$ is still negative :/ How does that make a difference? Could you explain through an example..
              $endgroup$
              – Abcd
              Feb 8 '18 at 12:27











              1












              $begingroup$

              This is not a complete answer, but rather a response to the following doubt:




              Now why do we add π to the expression when we are already having a positive argument ...




              Actually, we subtract $pi.$ But how are we to understand intuitively that this is something we might want to do?



              As you observed, under the given conditions on $x$ and $y,$ we know that
              $frac{x+y}{1-xy} > 0$ and therefore
              $tan^{-1}left(frac{x+y}{1-xy}right) > 0$ as well.



              But the given conditions include $x< 0$ and $y< 0$, from which it follows that
              $tan^{-1} x < 0,$ that $tan^{-1} y < 0,$ and that
              $tan^{-1} x + tan^{-1} y < 0.$



              We cannot have an equation with a negative number on the left and a positive number on the right, can we?
              But we can add or subtract something on the left or right in order to make the two sides equal after all.
              What is to be shown then is that the thing to add or subtract is a constant for all $x$ and $y$ that satisfy the given conditions,
              and that subtracting the particular constant $pi$ from the right side will make the equation satisfied.






              share|cite|improve this answer









              $endgroup$


















                1












                $begingroup$

                This is not a complete answer, but rather a response to the following doubt:




                Now why do we add π to the expression when we are already having a positive argument ...




                Actually, we subtract $pi.$ But how are we to understand intuitively that this is something we might want to do?



                As you observed, under the given conditions on $x$ and $y,$ we know that
                $frac{x+y}{1-xy} > 0$ and therefore
                $tan^{-1}left(frac{x+y}{1-xy}right) > 0$ as well.



                But the given conditions include $x< 0$ and $y< 0$, from which it follows that
                $tan^{-1} x < 0,$ that $tan^{-1} y < 0,$ and that
                $tan^{-1} x + tan^{-1} y < 0.$



                We cannot have an equation with a negative number on the left and a positive number on the right, can we?
                But we can add or subtract something on the left or right in order to make the two sides equal after all.
                What is to be shown then is that the thing to add or subtract is a constant for all $x$ and $y$ that satisfy the given conditions,
                and that subtracting the particular constant $pi$ from the right side will make the equation satisfied.






                share|cite|improve this answer









                $endgroup$
















                  1












                  1








                  1





                  $begingroup$

                  This is not a complete answer, but rather a response to the following doubt:




                  Now why do we add π to the expression when we are already having a positive argument ...




                  Actually, we subtract $pi.$ But how are we to understand intuitively that this is something we might want to do?



                  As you observed, under the given conditions on $x$ and $y,$ we know that
                  $frac{x+y}{1-xy} > 0$ and therefore
                  $tan^{-1}left(frac{x+y}{1-xy}right) > 0$ as well.



                  But the given conditions include $x< 0$ and $y< 0$, from which it follows that
                  $tan^{-1} x < 0,$ that $tan^{-1} y < 0,$ and that
                  $tan^{-1} x + tan^{-1} y < 0.$



                  We cannot have an equation with a negative number on the left and a positive number on the right, can we?
                  But we can add or subtract something on the left or right in order to make the two sides equal after all.
                  What is to be shown then is that the thing to add or subtract is a constant for all $x$ and $y$ that satisfy the given conditions,
                  and that subtracting the particular constant $pi$ from the right side will make the equation satisfied.






                  share|cite|improve this answer









                  $endgroup$



                  This is not a complete answer, but rather a response to the following doubt:




                  Now why do we add π to the expression when we are already having a positive argument ...




                  Actually, we subtract $pi.$ But how are we to understand intuitively that this is something we might want to do?



                  As you observed, under the given conditions on $x$ and $y,$ we know that
                  $frac{x+y}{1-xy} > 0$ and therefore
                  $tan^{-1}left(frac{x+y}{1-xy}right) > 0$ as well.



                  But the given conditions include $x< 0$ and $y< 0$, from which it follows that
                  $tan^{-1} x < 0,$ that $tan^{-1} y < 0,$ and that
                  $tan^{-1} x + tan^{-1} y < 0.$



                  We cannot have an equation with a negative number on the left and a positive number on the right, can we?
                  But we can add or subtract something on the left or right in order to make the two sides equal after all.
                  What is to be shown then is that the thing to add or subtract is a constant for all $x$ and $y$ that satisfy the given conditions,
                  and that subtracting the particular constant $pi$ from the right side will make the equation satisfied.







                  share|cite|improve this answer












                  share|cite|improve this answer



                  share|cite|improve this answer










                  answered Jan 13 at 18:35









                  David KDavid K

                  53.9k342116




                  53.9k342116






























                      draft saved

                      draft discarded




















































                      Thanks for contributing an answer to Mathematics Stack Exchange!


                      • Please be sure to answer the question. Provide details and share your research!

                      But avoid



                      • Asking for help, clarification, or responding to other answers.

                      • Making statements based on opinion; back them up with references or personal experience.


                      Use MathJax to format equations. MathJax reference.


                      To learn more, see our tips on writing great answers.




                      draft saved


                      draft discarded














                      StackExchange.ready(
                      function () {
                      StackExchange.openid.initPostLogin('.new-post-login', 'https%3a%2f%2fmath.stackexchange.com%2fquestions%2f1837410%2finverse-trigonometric-function-identity-doubt-tan-1x-tan-1y-pi-tan%23new-answer', 'question_page');
                      }
                      );

                      Post as a guest















                      Required, but never shown





















































                      Required, but never shown














                      Required, but never shown












                      Required, but never shown







                      Required, but never shown

































                      Required, but never shown














                      Required, but never shown












                      Required, but never shown







                      Required, but never shown







                      Popular posts from this blog

                      Mario Kart Wii

                      What does “Dominus providebit” mean?

                      Antonio Litta Visconti Arese